We have your learning goals covered with Spring and Summer courses available. Enroll today!

G
Topic
First Poster
Last Poster
Solve this hard problem:
slimshadyyy.3.60   2
N 14 minutes ago by Alex-131
Let a,b,c be positive real numbers such that x +y+z = 3. Prove that
yx^3 +zy^3+xz^3+9xyz≤ 12.
2 replies
slimshadyyy.3.60
2 hours ago
Alex-131
14 minutes ago
Counting Numbers
steven_zhang123   0
an hour ago
Source: China TST 2001 Quiz 8 P3
Let the decimal representations of numbers $A$ and $B$ be given as: $A = 0.a_1a_2\cdots a_k > 0$, $B = 0.b_1b_2\cdots b_k > 0$ (where $a_k, b_k$ can be 0), and let $S$ be the count of numbers $0.c_1c_2\cdots c_k$ such that $0.c_1c_2\cdots c_k < A$ and $0.c_kc_{k-1}\cdots c_1 < B$ ($c_k, c_1$ can also be 0). (Here, $0.c_1c_2\cdots c_r (c_r \neq 0)$ is considered the same as $0.c_1c_2\cdots c_r0\cdots0$).

Prove: $\left| S - 10^k AB \right| \leq 9k.$
0 replies
steven_zhang123
an hour ago
0 replies
Perfect Numbers
steven_zhang123   0
an hour ago
Source: China TST 2001 Quiz 8 P2
If the sum of all positive divisors (including itself) of a positive integer $n$ is $2n$, then $n$ is called a perfect number. For example, the sum of the positive divisors of 6 is $1 + 2 + 3 + 6 = 2 \times 6$, hence 6 is a perfect number.
Prove: There does not exist a perfect number of the form $p^a q^b r^c$, where $a, b, c$ are positive integers, and $p, q, r$ are odd primes.
0 replies
steven_zhang123
an hour ago
0 replies
Roots of unity
steven_zhang123   0
an hour ago
Source: China TST 2001 Quiz 8 P1
Let $k, n$ be positive integers, and let $\alpha_1, \alpha_2, \ldots, \alpha_n$ all be $k$-th roots of unity, satisfying:
\[
\alpha_1^j + \alpha_2^j + \cdots + \alpha_n^j = 0 \quad \text{for any } j (0 < j < k).
\]Prove that among $\alpha_1, \alpha_2, \ldots, \alpha_n$, each $k$-th root of unity appears the same number of times.
0 replies
steven_zhang123
an hour ago
0 replies
Graph Theory Test in China TST (space stations again)
steven_zhang123   0
an hour ago
Source: China TST 2001 Quiz 7 P3
MO Space City plans to construct $n$ space stations, with a unidirectional pipeline connecting every pair of stations. A station directly reachable from station P without passing through any other station is called a directly reachable station of P. The number of stations jointly directly reachable by the station pair $\{P, Q\}$ is to be examined. The plan requires that all station pairs have the same number of jointly directly reachable stations.

(1) Calculate the number of unidirectional cyclic triangles in the space city constructed according to this requirement. (If there are unidirectional pipelines among three space stations A, B, C forming $A \rightarrow B \rightarrow C \rightarrow A$, then triangle ABC is called a unidirectional cyclic triangle.)

(2) Can a space city with $n$ stations meeting the above planning requirements be constructed for infinitely many integers $n \geq 3$?
0 replies
steven_zhang123
an hour ago
0 replies
How many cases did you check?
avisioner   16
N an hour ago by eezad3
Source: 2023 ISL N2
Determine all ordered pairs $(a,p)$ of positive integers, with $p$ prime, such that $p^a+a^4$ is a perfect square.

Proposed by Tahjib Hossain Khan, Bangladesh
16 replies
avisioner
Jul 17, 2024
eezad3
an hour ago
A and B play a game
EthanWYX2009   2
N an hour ago by steven_zhang123
Source: 2025 TST 23
Let \( n \geq 2 \) be an integer. Two players, Alice and Bob, play the following game on the complete graph \( K_n \): They take turns to perform operations, where each operation consists of coloring one or two edges that have not been colored yet. The game terminates if at any point there exists a triangle whose three edges are all colored.

Prove that there exists a positive number \(\varepsilon\), Alice has a strategy such that, no matter how Bob colors the edges, the game terminates with the number of colored edges not exceeding
\[
\left( \frac{1}{4} - \varepsilon \right) n^2 + n.
\]
2 replies
EthanWYX2009
Yesterday at 2:49 PM
steven_zhang123
an hour ago
Graph again
steven_zhang123   0
an hour ago
Source: China TST 2001 Quiz 7 P2
Let \(L_3 = \{3\}\), \(L_n = \{3, 4, \ldots, h\}\) (where \(h > 3\)). For any given integer \(n \geq 3\), consider a graph \(G\) with \(n\) vertices that contains a Hamiltonian cycle \(C\) and has more than \(\frac{n^2}{4}\) edges. For which lengths \(l \in L_n\) must the graph \(G\) necessarily contain a cycle of length \(l\)?
0 replies
steven_zhang123
an hour ago
0 replies
Why are there so many Graphs in China TST 2001?
steven_zhang123   0
an hour ago
Source: China TST 2001 Quiz 7 P1
Let $k$ be a given integer, $3 < k \leq n$. Consider a graph $G$ with $n$ vertices satisfying the condition: for any two non-adjacent vertices $x$ and $y$ in graph $G$, the sum of their degrees must satisfy $d(x) + d(y) \geq k$. Please answer the following questions and prove your conclusions.

(1) Suppose the length of the longest path in graph $G$ is $l$ satisfying the inequality $3 \leq l < k$, does graph $G$ necessarily contain a cycle of length $l+1$? (The length of a path or cycle refers to the number of edges that make up the path or cycle.)

(2) For the case where $3 < k \leq n-1$ and graph $G$ is connected, can we determine that the length of the longest path in graph $G$, $l \geq k$?

(3) For the case where $3 < k = n-1$, is it necessary for graph $G$ to have a path of length $n-1$ (i.e., a Hamiltonian path)?
0 replies
steven_zhang123
an hour ago
0 replies
2025 TST 22
EthanWYX2009   2
N an hour ago by DottedCaculator
Source: 2025 TST 22
Let \( A \) be a set of 2025 positive real numbers. For a subset \( T \subseteq A \), define \( M_T \) as the median of \( T \) when all elements of \( T \) are arranged in increasing order, with the convention that \( M_\emptyset = 0 \). Define
\[
P(A) = \sum_{\substack{T \subseteq A \\ |T| \text{ odd}}} M_T, \quad Q(A) = \sum_{\substack{T \subseteq A \\ |T| \text{ even}}} M_T.
\]Find the smallest real number \( C \) such that for any set \( A \) of 2025 positive real numbers, the following inequality holds:
\[
P(A) - Q(A) \leq C \cdot \max(A),
\]where \(\max(A)\) denotes the largest element in \( A \).
2 replies
EthanWYX2009
Yesterday at 2:50 PM
DottedCaculator
an hour ago
The Quest for Remainder
steven_zhang123   0
an hour ago
Source: China TST 2001 Quiz 6 P3
Given sets $A = \{1, 4, 5, 6, 7, 9, 11, 16, 17\}$, $B = \{2, 3, 8, 10, 12, 13, 14, 15, 18\}$, if a positive integer leaves a remainder (the smallest non-negative remainder) that belongs to $A$ when divided by 19, then that positive integer is called an $\alpha$ number. If a positive integer leaves a remainder that belongs to $B$ when divided by 19, then that positive integer is called a $\beta$ number.
(1) For what positive integer $n$, among all its positive divisors, are the numbers of $\alpha$ divisors and $\beta$ divisors equal?
(2) For which positive integers $k$, are the numbers of $\alpha$ divisors less than the numbers of $\beta$ divisors? For which positive integers $l$, are the numbers of $\alpha$ divisors greater than the numbers of $\beta$ divisors?
0 replies
steven_zhang123
an hour ago
0 replies
Cyclic ine
m4thbl3nd3r   9
N Mar 24, 2025 by Nguyenhuyen_AG
Let $a,b,c>0$ such that $a+b+c=3$. Prove that $$a^3b+b^3c+c^3a+9abc\le 12$$May everyone not to upload solutions on this problem anymore until May 15/2025, this is an active problem on Mathematical Reflection! (Thank you Victoria_Discalceata1)
9 replies
m4thbl3nd3r
Mar 22, 2025
Nguyenhuyen_AG
Mar 24, 2025
Cyclic ine
G H J
G H BBookmark kLocked kLocked NReply
The post below has been deleted. Click to close.
This post has been deleted. Click here to see post.
m4thbl3nd3r
271 posts
#1
Y by
Let $a,b,c>0$ such that $a+b+c=3$. Prove that $$a^3b+b^3c+c^3a+9abc\le 12$$May everyone not to upload solutions on this problem anymore until May 15/2025, this is an active problem on Mathematical Reflection! (Thank you Victoria_Discalceata1)
This post has been edited 3 times. Last edited by m4thbl3nd3r, Mar 23, 2025, 2:04 PM
Z K Y
The post below has been deleted. Click to close.
This post has been deleted. Click here to see post.
JK1603JK
28 posts
#2
Y by
Proof.

The inequality is rewritten as 3\left(a^{2}b+b^{2}c+c^{2}a\right)-\left(ab+bc+ca\right)^{2}+12abc-12\le 0.

Now, we may use a^{2}b+b^{2}c+c^{2}a\le 4-abc.
Hence, it's enough to prove (ab+bc+ca)^2\ge 9abc=3abc(a+b+c),

which is obviously true. Equality holds iff a=b=c=1

Remark.

For k\ge k_0=\dfrac{1419}{256} then
a^3b+b^3c+c^3a+k\cdot abc\le 3+k holds \forall a,b,c\ge 0: a+b+c=3.
Equality cases: (a,b,c)\sim(1,1,1) or (a,b,c)\sim\left(\dfrac{3}{4},0,\dfrac{9}{4}\right).
Z K Y
The post below has been deleted. Click to close.
This post has been deleted. Click here to see post.
m4thbl3nd3r
271 posts
#3
Y by
JK1603JK wrote:
Proof.

The inequality is rewritten as $$3\left(a^{2}b+b^{2}c+c^{2}a\right)-\left(ab+bc+ca\right)^{2}+12abc-12\le 0.$$
Now, we may use $$a^{2}b+b^{2}c+c^{2}a\le 4-abc.$$Hence, it's enough to prove $$(ab+bc+ca)^2\ge 9abc=3abc(a+b+c),$$
which is obviously true. Equality holds iff a=b=c=1

Remark.

For $k\ge k_0=\dfrac{1419}{256}$ then
$$a^3b+b^3c+c^3a+k\cdot abc\le 3+k holds \forall a,b,c\ge 0: a+b+c=3.$$Equality cases: $(a,b,c)\sim(1,1,1) or (a,b,c)\sim\left(\dfrac{3}{4},0,\dfrac{9}{4}\right).$
LaTeXed :D
Z K Y
The post below has been deleted. Click to close.
This post has been deleted. Click here to see post.
jokehim
1027 posts
#4
Y by
Also, $$a^3b+b^3c+c^3a+6abc\le 9$$holds for all $a,b,c\in\mathbb{R}$ with $a+b+c=3.$
Z K Y
The post below has been deleted. Click to close.
This post has been deleted. Click here to see post.
Nguyenhuyen_AG
3298 posts
#5 • 1 Y
Y by ehuseyinyigit
m4thbl3nd3r wrote:
Let $a,b,c>0$ such that $a+b+c=3$. Prove that $$a^3b+b^3c+c^3a+9abc\le 12$$
Assume that $b = \text{mid}\{a,b,c\},$ then $(a-b)(b-c) \geqslant 0.$ We have
\[a^3b+b^3c+c^3a \leqslant a^3b+b^3c+c^3a + c(b+c)(a-b)(b-c) = b(c^3+abc+a^3).\]Thus
\[\begin{aligned}
a^3b+b^3c+c^3a+9abc &\leqslant b(c^3+abc+a^3)+9abc \\
& = b[(c+a)^3-3ca(c+a)+(b+9)ca] \\
& = b[(3-b)^3-3ca(3-b)+(b+9)ca] \\
& = b[4bca-(b-3)^3] \\
& \leqslant b[b(c+a)^2-(b-3)^3] \\
& \leqslant 3b(b-3)^2.
\end{aligned}\]It remains to prove that
\[b(b-3)^2 \leqslant 4,\]or
\[(4-b)(b-1)^2 \geqslant 0.\]Which is true.
Z K Y
The post below has been deleted. Click to close.
This post has been deleted. Click here to see post.
sqing
41310 posts
#6
Y by
Let $ a,b,c\geq 0 $ and $a+b+c=3$. Prove that$$a^3b+b^3c+c^3a+\frac{1419}{256}abc\le\frac{2187}{256}$$Equality holds when $ a=b=c=1 $ or $ a=0,b=\frac{9}{4},c=\frac{3}{4} $ or $ a=\frac{3}{4} ,b=0,c=\frac{9}{4} $
or $ a=\frac{9}{4} ,b=\frac{3}{4},c=0. $
*
Z K Y
The post below has been deleted. Click to close.
This post has been deleted. Click here to see post.
Quantum-Phantom
245 posts
#7 • 1 Y
Y by ehuseyinyigit
According to $a+b+c=3$, it suffices to show that
\[4(a+b+c)^3\ge27\left(a^3b+b^3c+c^3a\right)+81abc(a+b+c),\tag{1}\]or
\[\sum_{\rm cyc}a^4+6\sum_{\rm cyc}a^2b^2-\frac{33}4\sum_{\rm cyc}a^2bc\ge\frac{11}4\sum_{\rm cyc}a^3b-4\sum_{\rm cyc}a^3c.\tag{2}\]
It is a result from Vasile Cirtoaje that the inequality
\[\sum_{\rm cyc}a^4+r\sum_{\rm cyc}a^2b^2+(p+q-r-1)\sum_{\rm cyc}a^2bc\ge p\sum_{\rm cyc}a^3b+q\sum_{\rm cyc}a^3c\]holds for all real numbers $a$, $b$, $c$ if and only if $3(1+r)\ge p^2+pq+q^2$. Since
\[3(1+6)=21\ge\left(\frac{11}4\right)^2+(-4)^2+\frac{11}4(-4)=\frac{201}{16},\]then (2) indeed holds. We are done.

Remark. The proof that Vasile Cirtoaje used to justify his result leads to an SOS expression for (1), as follows.
\[\frac18\sum_{\rm cyc}\left(4 a^2-2ab+9ac-7bc-4c^2\right)^2+\frac{45}8\sum_{\rm cyc}a^2(b-c)^2\ge0.\]
Z K Y
The post below has been deleted. Click to close.
This post has been deleted. Click here to see post.
Victoria_Discalceata1
743 posts
#8
Y by
No source given for the OP? Ok, here it goes.
It is Math Reflections S694, an active problem. Its deadline is May 15, 2025.
Z K Y
The post below has been deleted. Click to close.
This post has been deleted. Click here to see post.
m4thbl3nd3r
271 posts
#9
Y by
Victoria_Discalceata1 wrote:
No source given for the OP? Ok, here it goes.
It is Math Reflections S694, an active problem. Its deadline is May 15, 2025.

Sorry for that, I don't read much on that Magazine. This is the problem on the test my teacher gave us. May everyone not to post solution on this problem anymore until May 15/2025
This post has been edited 1 time. Last edited by m4thbl3nd3r, Mar 23, 2025, 2:00 PM
Z K Y
The post below has been deleted. Click to close.
This post has been deleted. Click here to see post.
Nguyenhuyen_AG
3298 posts
#11
Y by
jokehim wrote:
Also, $$a^3b+b^3c+c^3a+6abc\le 9$$holds for all $a,b,c\in\mathbb{R}$ with $a+b+c=3.$
See here: https://artofproblemsolving.com/community/c6h432676p2640742
Z K Y
N Quick Reply
G
H
=
a